Telescoping Sum Convergence: Explained and Solved with Examples | Homework Help

  • Thread starter Thread starter Fuzedmind
  • Start date Start date
  • Tags Tags
    Issues Sum
Click For Summary
The discussion focuses on solving a series expressed as a telescoping sum, specifically the series 2/(n^2 + 4n + 3). The initial attempt at partial fraction decomposition was incorrect, but a correct form was identified as 1/(n+1) - 1/(n+3), which allows for cancellation of terms. Participants demonstrated how to write out the first several terms to visualize the cancellation process. The key to finding convergence lies in identifying which terms remain uncancelled as the series progresses. The conversation emphasizes the importance of correctly applying partial fractions to facilitate the telescoping nature of the series.
Fuzedmind
Messages
9
Reaction score
0

Homework Statement



The problem asks me to express the sum of the series as a telescoping sum, then find whether it is convergent or divergent. Ok, I get that and how it works and all, but the examples they give in the book are stupid and i on spring break this week so no office hours for professors.

Homework Equations


Here it is:

2/(n^2 + 4n + 3)

I know, easy, but I don't get how to do it...the easy ones stump me.

The Attempt at a Solution



I rewrote it like this:
(1/2)(2/n+3) - 2/n+1)

But the terms do not cancel when I do this. Plus it is an even question so I do not know the solution.
 
Physics news on Phys.org
Fuzedmind said:
..but the examples they give in the book are stupid and i on spring break this week

I can tell :smile:

You didn't do the partial fractions right; This is apparent by plugging in n=0 to the original and what you got: \frac{2}{n^2+4n+3}=\frac{1}{n+1}-\frac{1}{n+3}. These terms WILL cancel at some point. Write out the first 5 or so terms of the series and you will see this.
 
Well I did that, and they started cancelling, and I got

(1 - 1/3) + (1/2 - 1/4) + (1/3 - 1/5) + (1/4 - 1/6) + (1/5 - 1/7)

I canceled the 1/3, 1/4, and the 1/5 out, but where do I go from there?

Sorry I am kind of retarted
 
Now write a few more terms and cancel the 1/6 and 1/7. What terms don't cancel? I kind of have faith that you aren't THAT retarded.
 
What you want to do is make sure that, for each "-1/(n+1)", there exist an m so that its "1/(m+3)" cancels it. That is, given an integer n, what m will make 1/(m+3)= 1/(n+1)?
 
Question: A clock's minute hand has length 4 and its hour hand has length 3. What is the distance between the tips at the moment when it is increasing most rapidly?(Putnam Exam Question) Answer: Making assumption that both the hands moves at constant angular velocities, the answer is ## \sqrt{7} .## But don't you think this assumption is somewhat doubtful and wrong?

Similar threads

  • · Replies 7 ·
Replies
7
Views
2K
  • · Replies 4 ·
Replies
4
Views
1K
  • · Replies 23 ·
Replies
23
Views
3K
Replies
7
Views
2K
Replies
2
Views
5K
  • · Replies 4 ·
Replies
4
Views
2K
Replies
3
Views
2K
  • · Replies 2 ·
Replies
2
Views
2K
  • · Replies 7 ·
Replies
7
Views
1K
  • · Replies 5 ·
Replies
5
Views
2K